Difference between revisions of "2023 WSMO Speed Round Problems/Problem 6"
(Created page with "==Problem== Let <math>ABC</math> be an equilateral triangle of side length <math>6.</math> Points <math>A_1,A_2,A_3,B_1,B_2,B_3,C_1,C_2,C_3</math> are chosen such that <math>...") |
|||
(One intermediate revision by the same user not shown) | |||
Line 4: | Line 4: | ||
==Solution== | ==Solution== | ||
+ | Let <math>[XYZ]</math> denote the area of triangle <math>XYZ</math>. We have | ||
+ | <cmath>\begin{align*} | ||
+ | [A_iB_jC_k] &= [ABC]-[AB_jC_k]-[BC_kA_i]-[CA_iB_j]\implies\\ | ||
+ | &= 6^2\cdot\frac{\sqrt{3}}{4}-\frac{1}{2}\sin(60^{\circ})(AB_j)(AC_k)-\frac{1}{2}\sin(60^{\circ})(BC_k)(BA_i)-\frac{1}{2}\sin(60^{\circ})(CA_i)(CB_j) \\ | ||
+ | &= 9\sqrt{3}-\frac{\sqrt{3}}{4}\left((AB_j)(AC_k)+(BC_k)(BA_i)+(CA_i)(CB_j)\right).\\ | ||
+ | \end{align*}</cmath> | ||
+ | Let <math>AB_j=c</math>, <math>BC_k=a,</math> and <math>CA_i=b</math>. This directly implies that <math>B_jC=6-c</math>, <math>C_kA=6-a</math>, and <math>A_iB=6-b</math>. So, we have | ||
+ | <cmath>\begin{align*} | ||
+ | [A_iB_jC_k] &= 9\sqrt{3}-\frac{\sqrt{3}}{4}\left(c(6-a)+a(6-b)+b(6-c)\right)\\ | ||
+ | &=9\sqrt{3}-\frac{\sqrt{3}}{4}\left(6(a+b+c)-(ab+ac+bc)\right)\implies\\ | ||
+ | \mathbb{E}([A_iB_jC_k)) &= 9\sqrt{3}-\frac{\sqrt{3}}{4}\left(6\cdot(\mathbb{E}(a)+\mathbb{E}(b)+\mathbb{E}(c))-(\mathbb{E}(ab)+\mathbb{E}(ac)+\mathbb{E}(bc)\right).\\ | ||
+ | \end{align*}</cmath> | ||
+ | We have <cmath>\mathbb{E}(a) = \mathbb{E}(b) = \mathbb{E}(c) = \frac{6}{2} = 3</cmath> and <cmath>\mathbb{E}(ab) = \mathbb{E}(ac) = \mathbb{E}(bc) = \left(\mathbb{E}(a)\right)^2 = 9.</cmath> Substituting, | ||
+ | <cmath>\begin{align*} | ||
+ | \mathbb{E}([A_iB_jC_k]) &= 9\sqrt{3}-\frac{\sqrt{3}}{4}\left(6\cdot\left(3+3+3\right)-\left(9+9+9\right)\right)\\ | ||
+ | &= 9\sqrt{3} - \frac{\sqrt{3}}{4}\left(54-27\right) = 9\sqrt{3}-\frac{27\sqrt{3}}{4}\\ | ||
+ | &=\frac{9\sqrt{3}}{4}\Rightarrow 9+3+4 = \boxed{16}. | ||
+ | \end{align*}</cmath> | ||
+ | |||
+ | ~pinkpig |
Latest revision as of 11:29, 12 September 2025
Problem
Let be an equilateral triangle of side length
Points
are chosen such that
divide
into four equal segments,
divide
into four equal segments, and
divide
into four equal segments. If
are chosen from the set
independently and randomly, the expected area of
is
for squarefree
and relatively prime positive integers
and
Find
Solution
Let denote the area of triangle
. We have
Let
,
and
. This directly implies that
,
, and
. So, we have
We have
and
Substituting,
~pinkpig